The Wronskian and diff.equations

  • Thread starter Thread starter Susanne217
  • Start date Start date
  • Tags Tags
    Wronskian
Click For Summary
The discussion revolves around demonstrating that a specific function y_2(t) is a solution to a given second-order differential equation, given a known solution y_1(t). The Wronskian is defined as w(y_1,y_2)(t) = c · e^(-∫p(t) dt), which is relevant for determining the relationship between the two solutions. The method of reduction of order is suggested as a technique to find y_2, involving substituting y_2 in terms of y_1 and a function F(t). Participants express confusion about the absence of this method in standard calculus texts, while also confirming its validity and relevance to the problem at hand. The conversation highlights the connection between the proposed solution and established methods in differential equations.
Susanne217
Messages
311
Reaction score
0
Hi

Homework Statement



I have differential equation

y'' + p(x)\cdot x' + q(x)y = 0 which have two solutions y_1(x) and y_2(x) where y_1(x) \neq 0


show that y_2(t) = y_1(t)\int_{t_0}^{t} \frac{1}{y_1(s)^2} e^{-\int_{t_0}^s p(r) dr} ds is also a solution.

Homework Equations



I know that wronskian for this is defined

w(y_1,y_2)(t) = c \cdot e^{-\int p(t) dt} according to Abels identity theorem.

The Attempt at a Solution



My question do I use this property of theorem and properties of the Wronskian to derive c?
 
Last edited:
Physics news on Phys.org
Susanne -

The solution specified for y2 is the general form of a second solution to second order differential equation when the roots to its characteristic equation are repeated. I am not sure what to do when you have functions as coefficients, but perhaps if you can show your second order equation can be reduced to a second order equation with constant coefficients (with repeated roots), then you may assume the solution y2 = F(t)*y1 (F a function to be determined). Twice differentiating y2 and substituting it back into your original second order equation, you should obtain a new equation in terms of F', and F''. Hence you may let g = F' and obtain a first order, separable equation. Solve for g, then back-substitute F' and integrate to solve for F. Finally, since y2 = F(t)*y1, and since F(t) is known, you should get the answer specified for y2. Hope this helps.

- Sam
 
Samuelb88 said:
Susanne -

The solution specified for y2 is the general form of a second solution to second order differential equation when the roots to its characteristic equation are repeated. I am not sure what to do when you have functions as coefficients, but perhaps if you can show your second order equation can be reduced to a second order equation with constant coefficients (with repeated roots), then you may assume the solution y2 = F(t)*y1 (F a function to be determined). Twice differentiating y2 and substituting it back into your original second order equation, you should obtain a new equation in terms of F', and F''. Hence you may let g = F' and obtain a first order, separable equation. Solve for g, then back-substitute F' and integrate to solve for F. Finally, since y2 = F(t)*y1, and since F(t) is known, you should get the answer specified for y2. Hope this helps.

- Sam

does this have any relation to reduction of order tecnique ?
 
Yes, I believe the name of the method I described above is reduction of order. Sorry, I forgot its name.
 
Samuelb88 said:
Yes, I believe the name of the method I described above is reduction of order. Sorry, I forgot its name.

Why this isn't in my Calculus book. I don't know :(

Found this is a different textbook, but thanks ;)

Found this note here:

http://en.wikipedia.org/wiki/Reduction_of_order

You seem to me more intelligent than me and therefore is the ansatz mentioned

y_2(x) = y_1(x) \cdot v(x) can this be describe as linearcombination of first solution and the abtr function v(x)??
 
Question: A clock's minute hand has length 4 and its hour hand has length 3. What is the distance between the tips at the moment when it is increasing most rapidly?(Putnam Exam Question) Answer: Making assumption that both the hands moves at constant angular velocities, the answer is ## \sqrt{7} .## But don't you think this assumption is somewhat doubtful and wrong?

Similar threads

Replies
2
Views
1K
  • · Replies 5 ·
Replies
5
Views
1K
Replies
1
Views
2K
  • · Replies 6 ·
Replies
6
Views
2K
  • · Replies 2 ·
Replies
2
Views
2K
  • · Replies 7 ·
Replies
7
Views
2K
  • · Replies 7 ·
Replies
7
Views
2K
  • · Replies 11 ·
Replies
11
Views
3K
Replies
2
Views
1K
  • · Replies 3 ·
Replies
3
Views
5K